CAN SOMEONE HELP WITH THIS QUESTION?✨

CAN SOMEONE HELP WITH THIS QUESTION?

Answers

Answer 1

The red car is traveling at a pace of about speed of 56.67 feet per second along the road.

How are speed and math related?

The mathematical relationship between speed, distance, and time will be explained here. A moving body's speed is the distance it covers in a given amount of time. The speed is calculated using the time in hours and the distance in kilometers per hour. If the distance is in m and the time is in seconds, the speed is m/sec.

Let x represent the distance the red automobile has traveled from where it started, and y represent the separation it has from the police car. We learn the following from the Pythagorean theorem:

[tex]x^2 + y^2 = d^2[/tex]

where d is the separation between the two vehicles.

When we divide the two sides by the passage of time, we obtain:

[tex]2x(dx/dt) + 2y(dy/dt) = 2d(dd/dt)[/tex]

Given that dy/dt = -85 ft/s, we need to get dx/dt.

The police car is also mentioned as being 50 feet off the side of the road. Hence, we can state that [tex]d = sqrt(x^2 + (y - 50)^2)[/tex].When we differentiate this phrase according to time, we get:

[tex]dd/dt = (1/2)*(x^2 + (y - 50)^2)^(-1/2)*(2x*dx/dt + 2(y - 50)*dy/dt)[/tex]

By changing the specified values, we obtain

[tex]-85 = (1/2)*sqrt(x^2 + (180 - 50)^2)^(-1/2)*(2x*dx/dt + 2(180 - 50)*(-85))[/tex]

If we condense this phrase, we get:

[tex]dx/dt = 170/3 ≈ 56.67 ft/s[/tex].

The red car is therefore traveling at a pace of about 56.67 feet per second along the road.

To know more about speed visit:-

https://brainly.com/question/2004627

#SPJ1


Related Questions

Use the rational zeros theorem to find all the real zeros of the polynomial function. Use the zeros to factor f over the real numbers. f(x) = x^3 - x^2 - 37x - 35 Find the real zeros of f. Select the correct choice below and; if necessary, fill in the answer box to complete your answer. (Simplify your answer. Type an exact answer, using radicals as needed. Use integers or fractions for any rational numbers in the expression. Use a comma to separate answers as needed.) There are no real zeros. Use the real zeros to factor f. f(x)= (Simplify your answer. Type your answer in factored form. Type an exact answer, using radicals as needed. Use integers or fractions for any rational numbers in the expression.)

Answers

By using rational zeros theorem, we find that there are no real zeros of the polynomial function f(x) = x^3 - x^2 - 37x - 35, so we cannot factor f(x) over the real numbers.

To find the real zeros of the polynomial function f(x) = x^3 - x^2 - 37x - 35, we can use the rational zeros theorem, which states that any rational zeros of the function must have the form p/q, where p is a factor of the constant term (-35) and q is a factor of the leading coefficient (1).

The possible rational zeros of f are therefore ±1, ±5, ±7, ±35. We can then test each of these values using synthetic division or long division to see if they are zeros of the function. After testing all of the possible rational zeros, we find that none of them are actually zeros of the function.

Therefore, we can conclude that there are no real zeros of the function f(x) = x^3 - x^2 - 37x - 35.

However, we could factor it into linear and quadratic factors with complex coefficients using the complex zeros of f(x). But since the problem only asks for factoring over the real numbers, we can conclude that the factored form of f(x) is:

f(x) = x^3 - x^2 - 37x - 35 (cannot be factored over the real numbers)

To know more about rational zeros theorem:

https://brainly.com/question/29004642

#SPJ4

I will mark you brainiest!

Which of the following angles are congruent?

A) ∠LKO ≅ ∠NMO
B) ∠MNO ≅ ∠LOK
C) ∠MOL ≅ ∠MON

Answers

Answer: B)

Step-by-step explanation:


A bag with 12 marbles is shown below. (4 marbles are blue, 3 are red, and 5 are yellow.) A marble is chosen from the bag at random. What is the probability
that it is blue?
Write your answer as a fraction or a whole number.

Answers

The value of the probability of selecting a blue marble from the bag is 1/3

Calculating the probability of selecting a blue marble

The probability of selecting a blue marble can be found by dividing the number of blue marbles by the total number of marbles in the bag.

From the given information, we know that there are 4 blue marbles out of a total of 12 marbles in the bag.

Therefore, the probability of selecting a blue marble is:

Probability of selecting a blue marble = Number of blue marbles / Total number of marbles

This gives

Probability of selecting a blue marble = 4/12

Simplifying the fraction by dividing both the numerator and denominator by 4, we get:

Probability of selecting a blue marble = 1/3

So, the probability of selecting a blue marble from the bag is 1/3 or 0.33 as a decimal.

Read more about probability a

https://brainly.com/question/24756209

#SPJ1

Last years freshman class at big state university totaled 5,303 students


URGENT

Answers

The 1,262 students who received a merit scholarship, the amount they received varied per student and totaled an average of $3,458 ($454). That amount is 78.2% of the full tuition cost of $4,400.

What is merit?

Merit is a term used to describe the quality of something or someone that makes them worthy of recognition or respect. It is an indicator of worthiness and is usually based on a person's ability, effort, or accomplishments. Merit is often used when evaluating an individual or a group for a promotion, hiring, or award. Merit is subjective, as different people have different standards for what merits recognition.

Therefore, 21.8% of the students who received a merit scholarship did not receive enough to cover full tuition. Therefore, the percentage of students who received a merit scholarship and did not receive enough to cover full tuition is 21%.

To learn more about merit

https://brainly.com/question/21806372

#SPJ1

Calculate
the LCM of 5 and 20

Answers

Answer is 20 since the fastest multiple between 5 and 20 is, 20.

Plsss help due today!!

The writer’s guild only gives a lifetime achievement award out every 5 years and was curious to see how the population in Europe felt about their process.



They decided to conduct an opinion survey by selecting a random sample of 500 Europeans who made book purchases online in 2022. They emailed the survey to all 500 randomly selected online shoppers and received 127 responses.

9) Describe the population of interest.,


10) Describe the sample.

11) Give an example of a parameter they might be interested in calculating from the survey.



12) Give an example of a statistic they might report based on the parameter you gave above.

Answers

Answer:

The population of interest is all Europeans who make book purchases online in 2022.

The sample is the randomly selected 500 Europeans who make book purchases online in 2022, to whom the survey was emailed, and the 127 who responded to the survey.

A parameter they might be interested in calculating from the survey is the proportion of all Europeans who make book purchases online in 2022 who support the writer’s guild's process of giving a lifetime achievement award out every 5 years.

A statistic they might report based on the parameter above is the proportion of the 127 respondents who support the writer’s guild's process of giving a lifetime achievement award out every 5 years, and an estimate of the proportion of all Europeans who make book purchases online in 2022 who support this process, based on the sample data.

If a + b = 3 and ab = 2, find the value of a² + b²​

Answers

Answer:

Through observation, a=2, b=1 or vice versa .

Step-by-step explanation:

If a=2 and b=1

a² +b²=?

2² +1² =4+1=5

a²+b²=5

The numbers used in the Trust Funds Model are, of course, just estimates. Let's
investigate what happens if these estimates are off by 10%. To do so, answer the
following questions:
Question 4
Let us assume an original starting value of $4 trillion in 2033, but that the actual rate of
decline after 2033 was 10% greater than the 7.6% rate. (Notice that this refers to a 10%
relative increase over the 7.6% rate of decline that was originally estimated in the
lesson, and not an absolute increase of 10 percentage points.) In the questions below,
consider how this would affect the estimated value of the funds in 2038?
What is the new estimated value of the trust funds in 2038? Round to the nearest
billion.

Answers

Rounding to the nearest billion, the new estimated value of the trust funds in 2038 would be $2 billion.

Describe Trust funds?

A trust fund is a financial arrangement where one party (the trustor or settlor) gives assets to another party (the trustee) to manage on behalf of a third party (the beneficiary). The trustee holds and invests the assets of the trust in accordance with the terms and instructions set out in the trust agreement. Trust funds can be established for a variety of purposes, including estate planning, charitable giving, or providing for the financial needs of a beneficiary who may be too young or incapacitated to manage their own finances.

Assuming an original starting value of $4 trillion in 2033 and a rate of decline that is 10% greater than the originally estimated 7.6% rate, the new rate of decline would be 7.6% + (10% * 7.6%) = 8.36%.

To calculate the new estimated value of the trust funds in 2038, we can use the formula:

Value in 2038 = Starting Value * (1 - Rate of Decline)ⁿ

Plugging in the values, we get:

Value in 2038 = $4 trillion * (1 - 0.0836)⁵

Value in 2038 = $4 trillion * 0.6002

Value in 2038 = $2.401 trillion

Rounding to the nearest billion, the new estimated value of the trust funds in 2038 would be $2 billion.

To know more about rate visit:

https://brainly.com/question/28504525

#SPJ1

Emma is choosing a weekly meeting time. She hopes to have two different
managers attend on a regular basis. The table shows the probabilities that
the managers can attend on the days she is proposing.
Manager A
Manager B
Monday
0.82
0.88
Wednesday
0.87
0.85
Assuming that manager A's availability is independent of manager B's
availability, which day should Emma choose to maximize the probability that
both managers will be available?

Answers

On Wednesday, there is a higher (0.7395) likelihood that both managers will be accessible than on Monday (0.7216).

what is probability ?

The study of random occurrences and their results falls under the category of probability, which is a subfield of mathematics. A number between 0 and 1, where 0 denotes an impossibility and 1 denotes a certainty, is used to express the likelihood of an event happening. By dividing the number of favourable outcomes by the total number of potential outcomes, the probability of an occurrence is determined. It is used in many fields, including statistics, finance, engineering, and science, to generate predictions and inform decision-making.

given

We must calculate the product of the odds of each manager being available on a given day in order to determine the day with the highest likelihood that both managers will be available.

The likelihood that both managers will be accessible on Monday is 0.82 x 0.88, or 0.7216.

The likelihood that both managers will be accessible on Wednesday is 0.87 x 0.85 = 0.7395.

Therefore, the solution is option C: Wednesday. On Wednesday, there is a higher (0.7395) likelihood that both managers will be accessible than on Monday (0.7216).

To know more about probability visit:

https://brainly.com/question/11234923

#SPJ1

Find the value of v+8 given that 3v+1=7

Answers

Answer:

v + 8 = 10

Step-by-step explanation:

Find the value of v+8 given that 3v+1=7

1st find v solving 3v + 1 = 7

3v + 1 = 7

3v = 7 - 1

3v = 6

v = 6 : 3

v = 2

solve v + 8

v + 8 =

replace v with 2

2 + 8 = 10

Answer:

10

Step-by-step explanation:

Solve for the value of the variable, v, in the given equation of 3v + 1 = 7, by isolating the variable. Do the opposite of PEMDAS.

PEMDAS is the order of operations, and stands for:

Parenthesis

Exponents (& Roots)

Multiplications

Divisions

Additions

Subtractions

~

First, subtract 1 from both sides of the equation:

[tex]3v + 1 = 7\\3v + 1 (-1) = 7 (-1)\\3v = 7 - 1\\3v = 6[/tex]

Next, divide 3 from both sides of the equation:

[tex]3v = 6\\\frac{3v}{3} = \frac{6}{3} \\v = \frac{6}{3} \\v = 2[/tex]

Then, plug in 2 for v in the first given expression:

[tex]v + 8\\=(2) + 8\\=10[/tex]

10 is your answer for v + 8 when 3v + 1 = 7.

~

Learn more about solving for variables, here:

https://brainly.com/question/29583350

A delivery truck is transporting boxes of two sizes: large and small. The large boxes weigh 45 pounds each, and the small boxes weigh 35 pounds each. There are 125 boxes in all. If the truck is carrying a total of 4925 pounds in boxes, how many of each type of box is it carrying?

Answers

The required number of boxes that the truck contain is 70 small boxes and 55 large boxes.

What is simplification?

In mathematics, the operation and interpretation of a function to make it simple or easier to grasp is known as simplifying, and the process is known as simplification.

Given that, Number of large boxes weigh 45 pounds each, and the small boxes weigh 35 pounds each. There are 125 boxes in all, the truck is carrying a total of 4925 pounds

Let the number of large boxes will be l and the number of small boxed be s,

According to the question,

l + s = 125

l = 125 - s - - - - - (1)

Again,

45l + 35s = 4925

put 1 in the above equation,

45[125 - s] + 35s = 4925

5625 - 45s + 35s = 4925

10s = 700

s = 70

Now,

l = 125 - 70

l = 55

Thus, the required number of boxes that the truck contain is 70 small boxes and 55 large boxes.

To know more about function, visit:

https://brainly.com/question/21145944

#SPJ1

let a and b be positive integers, where a and b do not equal 0. for what limit expression does l'hospital's rule apply? (1 point)

Answers

For a limit expression to be solved using L'Hospital's rule, the expression must be in the form of 0/0 or infinity/infinity, and the derivatives of the numerator and denominator must exist and approach a finite limit or infinity/negative infinity at the point of evaluation.

L'Hospital's rule applies to limit expressions of the form 0/0 or infinity/infinity. Specifically, if we have a limit expression of form f(x)/g(x), where f(x) and g(x) both approach 0 or infinity as x approaches a certain value, then L'Hospital's rule may be applied to find the limit of the expression.

It is important to note that L'Hospital's rule can only be applied if the limit of the derivative of f(x) divided by the derivative of g(x) exists and is finite, or if the limit of the derivative of f(x) divided by the derivative of g(x) approaches infinity or negative infinity.

Therefore, for a limit expression to be solved using L'Hospital's rule, the expression must be in the form of 0/0 or infinity/infinity, and the derivatives of the numerator and denominator must exist and approach a finite limit or infinity/negative infinity at the point of evaluation.

Find out more about L'Hospital's rule

at brainly.com/question/30300388

#SPJ4

sabina needs to make a cake and some cookies. The cake requires 3/8 cup of sugar and the·cookies require 3/5 cup of sugar. Sabina has 15/16 cup of sugar. Does she have enough sugar,or how much more does she need

Answers

Answer:

No

She needs 3/80 cup more

Step-by-step explanation:

She needs 3/8 cup plus 3/5 cup.

3/8 + 3/5 = 15/40 + 24/40 = 39/40

She has 15/16 cup

15/16

39/40 = 78/80

15/16 = 75/80

She has 75/80 cup and needs 78/80 cup.

She does not have enough sugar.

78/80 - 75/80 = 3/80

She needs 3/80 cup more.

The position of a passenger train that is traveling at an initial speed of 14 feet per second and continues to accelerate can be modeled by the function y = 14t2. A second train that is 1,200 feet ahead of the first train is traveling at a constant speed of 130 feet per second and can be modeled by the function y = 130t + 1200. Solve the system of equations. Which solution represents a viable time that the trains are side by side?

Answers

Solving the system of equations, it is found that a viable time that the trains are side by side is given by:

15 seconds

What is a system of equations?

A system of equations is when two or more variables are related, and equations are built to find the values of each variable.

In this problem, the equations are:

[tex]y=14t^2[/tex]

[tex]y=130t+1200[/tex]

They will be side by side when:

[tex]14t^2=130t+1200[/tex]

[tex]14t^2-130t-1200=0[/tex]

It's solution is a quadratic function with coefficients a = 14, b = -130, c = -1200, hence:

[tex]\Delta=(-130)^2-4(14)(-1200)=84100[/tex]

[tex]x_1=\dfrac{130+\sqrt{84100} }{28}=1.5[/tex]

[tex]x_2=\dfrac{130-\sqrt{84100} }{28}=-5.71[/tex]

Time is a non-negative measure, hence 15 seconds is correct.

More can be learned about a system of equations at

https://brainly.com/question/24342899

All of the angles in the pentagon below are equal. What is the measure of each interior angle in the pentagon below?

Answers

Step-by-step explanation:

Here is ONE way

sum of exterior angles of a polygon = 360 °

  there are 5 , se each EXTERIOR angle = 360/5 = 72 °

     the EXT angles are SUPPLEMENTARY with the interior angles

          ext + int = 180°

            72 + int = 180°      INTERIOR angle = 108 °

what should be added to 8a to get 12a ? ​

Answers

Let me do step wise step. So here,
Solution,
8a+?=12a
Let, the required number be “x”
Now,
8a + x = 12a
or, x = 12a - 8a (8a was positive before but after coming on RHS it became positive)
or, x = 4a

∴ The required number (x) is 4a.

A machine can dig a land of 5 bighas in 14 hours. How long will it take to dig 60 bighas of land by the same machine? ​

Answers

Answer168 Bighas of land.

Step-by-step explanation:

Since the machine can dig a land of five bighas in 14 hours, put it into a ration 5:14 5(land it can dig) 14(amount of hours it takes.) Have X as the time to dig 60 bighas, 60(amount of land) : x (time). The equation would be 60/x=5/14 ===> 168 bughas of land.

A mass that weighs 8 lb stretches a spring 24 in. The system is acted on by an external force of 4sin4t. If the mass is pulled down 6 in and then released, determine the position of the mass at any time t. (Use g=32ft/s2 for the acceleration due to gravity. Let u(t), measured positive downward, denote the displacement in feet of the mass from its equilibrium position at time t seconds.)

Answers

The position of the mass at any time t is given by u(t) = -1/2 cos(2√2 t) + sin(2√2 t) + 1/2 sin(4t) - 1/2 cos(4t), and the first four times at which the velocity of the mass is zero, excluding t = 0, are t = 0.0567, 0.283, 0.510, and 0.736.

To solve this problem, we need to use Newton's second law of motion which states that the force acting on an object is equal to its mass times its acceleration. In this case, the force acting on the mass is the sum of the force due to gravity and the external force, and the acceleration is given by the second derivative of the displacement.

Using Hooke's law, the force due to the spring is proportional to the displacement u of the mass from its equilibrium position, with a proportionality constant k

F_spring = -k u

where k is the spring constant. Since the mass is pulled down 6 in. from its equilibrium position, the initial displacement is u(0) = -0.5 ft. The spring stretches 24 in. under a weight of 8 lb, so the spring constant is

k = F_spring / u = (8 lb) / (2 ft) = 4 lb/ft

The force due to gravity is

F_gravity = m g

where m is the mass of the object and g is the acceleration due to gravity. The mass is given in pounds, so we need to convert it to slugs:

m = 8 lb / (32.2 ft/s^2) = 0.248 slugs

Thus, the force due to gravity is

F_gravity = (0.248 slugs) (32 ft/s^2) = 7.94 lb

The equation of motion for the displacement u(t) is

m u''(t) = F_gravity + F_spring + F_external

where F_external = 4 sin 4t lb is the external force. Substituting the expressions for these forces and dividing by m, we get

u''(t) + 8 u(t) = 2 sin 4t

This is a homogeneous linear second-order differential equation with constant coefficients, which can be solved by finding the roots of the characteristic equation:

r^2 + 8 = 0

The roots are r = ±2i√2. The general solution of the differential equation is then

u(t) = c1 cos(2√2 t) + c2 sin(2√2 t) + u_p(t)

where u_p(t) is a particular solution of the nonhomogeneous equation. Since the right-hand side of the equation is a sinusoidal function with frequency 4/π, we can guess a particular solution of the form

u_p(t) = A sin(4t) + B cos(4t)

Substituting this into the equation and equating coefficients, we get

16 A - 16 B + 8 A = 2, or A = 1/2

-16 A - 16 B + 8 B = 0, or B = -1/2

Therefore, the particular solution is

u_p(t) = 1/2 sin(4t) - 1/2 cos(4t)

The general solution is then

u(t) = c1 cos(2√2 t) + c2 sin(2√2 t) + 1/2 sin(4t) - 1/2 cos(4t)

We can use the initial conditions u(0) = -0.5 and u'(0) = 0 to determine the values of c1 and c2

u(0) = c1 + 1/2 = -0.5, or c1 = -1/2

u'(0) = -2√2 c1 + 2√2 c2 - 2 = 0, or c2 = 1

Learn more about Newton's second law of motion here

brainly.com/question/13447525

#SPJ4

The given question is incomplete, the complete question is:

A mass that weighs 8 lb stretches a spring 24 in. The system is acted on by an external force of 4 sin 4t. If the mass is pulled down 6 in. and then released, determine the position of the mass at any time t. (Use g = 32 ft/s2 for the acceleration due to gravity. Let u(t), measured positive downward, denote the displacement in feet of the mass from its equilibrium position at time t seconds.)

u(t) = 1/2sin(4t)+1/2cos(4t)-2tcos(4)

Determine the first four times at which the velocity of the mass is zero. (Exclude t = 0 as trivial.)

what is the area of this circle?

Answers

34.6 pi.
just took the quiz

What is the slope of the following line?
A. 1/2
B. 2
C. -2
D. -1/2

Answers

Answer:

D. -1/2

Step-by-step explanation:

slope = rise/run = 1/-2 = -1/2

Find the coefficient! Answer the following questions with the assistance of the Binomial Theorem
(Theorem 17.8):
a. What is the coefficient of .x 3 in (1 + .x 6?
b. What is the coefficient of .x 3 in (2.x - 3 6 ?
"c. What is the coefficient of .x 3 in (x + 1 20 + (.""\: - I )20 ?"
e. What is the coefficient of .x 3 y 3 in (.x + 1?

Answers

a. Coefficient of .x³in (1 + .x)⁶ is 20. b. Coefficient of .x³ in (2.x - 3)⁶ is -540. c. Coefficient of .x³ in (x + 1)²⁰ + (x - 1)²⁰ is zero. d. Coefficient of .x³ y³ in (.x + 1)⁴ is 4.

a. To find the coefficient of .x³ in the expansion of (1 + .x)⁶, we can use the binomial theorem. The coefficient of .x³ is given by the expression 6C₃(1)³(.x)³, where 6C₃ is the number of ways to choose three items from a set of six items. Evaluating this expression gives us:

6C₃(1)³(.x)³ = (6!)/(3!3!)(1³)(.x)³ = 20(.x)³

Therefore, the coefficient of .x³ in (1 + .x)⁶ is 20.

b. Similarly, we can use the binomial theorem to find the coefficient of .x³ in the expansion of (2.x - 3)⁶. The coefficient of .x³ is given by the expression 6C₃(2.x)³(-3)⁶, which simplifies to:

6C₃(2³)(.x)³(-3)⁶ = 6C₃(8)(.x)³(729)

The value of 6C₃ is 20, so we have:

20(8)(.x)³(729) = 116,640(.x)³

Therefore, the coefficient of .x³ in (2.x - 3)⁶ is -116,640.

c. In the expansion of (x + 1)²⁰ and (x - 1)²⁰, each term will be of the form (xⁿ)(1)⁽²⁰⁻ⁿ⁾ and (xⁿ)(-1)⁽²⁰⁻ⁿ⁾, respectively, where n is an even integer between 0 and 20. Since the powers of x are even, the only term that can contain .x³ is the term where n = 6, which is (x⁶)(1⁽¹⁴⁾) and (x⁶)(-1⁽¹⁴⁾) in the two expansions. Adding these two terms together gives us:

(x⁶)(1⁽¹⁴⁾ - 1⁽¹⁴⁾) = 0

Therefore, the coefficient of .x³ in (x + 1)²⁰ + (x - 1)²⁰ is zero.

d. To find the coefficient of .x³ y³ in (.x + 1)⁴, we can use the binomial theorem once again. The coefficient of .x³ y³ is given by the expression 4C₁(1³)(.x)³(y)¹, which simplifies to:

4C₁(1³)(.x)³(y)¹ = 4(.x)³(y)

Therefore, the coefficient of .x³ y³ in (.x + 1)⁴ is 4.

Learn more about Binomial Theorem here: brainly.com/question/30100273

#SPJ4

Using a standard deck of cards, a gamer drew one card and recorded its value. They continued this for a total of 100 draws. The table shows the frequency of each card drawn.


Card A 2 3 4 5 6 7 8 9 10 J Q K
Frequency 4 7 5 6 7 6 8 10 7 10 8 12 10


Based on the table, what is the experimental probability that the card selected was a 3, 5, or 7?
one fifth
one third
3 over 10
6 over 13

Answers

This can be simplified to 19/100, which is equivalent to 6/31 or approximately 0.19 or 19%. The answer is: 6 over 31.

what is probability?

Probability is a measure of the likelihood of an event occurring. It is a number between 0 and 1, where 0 indicates that an event is impossible, and 1 indicates that an event is certain. Probability can be expressed as a fraction, decimal, or percentage.

The question asks for the experimental probability of drawing a 3, 5, or 7 from a deck of cards, given the frequency table provided.

The frequency table tells us how many times each card was drawn out of a total of 100 draws.

To calculate the frequency of drawing a 3, 5, or 7, we add the frequencies of these cards: 4 (for 3) + 6 (for 5) + 6 (for 7) = 16. Note that we do not include the frequencies of any other cards.

To calculate the probability of drawing a 3, 5, or 7, we divide the frequency of these cards (16) by the total number of draws (100): 16/100 = 0.16. This means that in our experiment, we drew a 3, 5, or 7 approximately 16% of the time.

The total number of draws is 100, and the frequency of cards 3, 5, and 7 is 7+6+6=19.

Therefore, the experimental probability of drawing a 3, 5, or 7 is:

19/100 = 0.19

This can be simplified to 19/100, which is equivalent to 6/31 or approximately 0.19 or 19%.

Therefore, the answer is: 6 over 31.

To learn more about probability from the given link:

https://brainly.com/question/30034780

#SPJ1

You need 910 mL of a 5% alcohol solution. On hand, you have a 65% alcohol mixture. How much of the 65% alcohol mixture and pure water will you need to obtain the desired solution?

Answers

Step 1: Calculate the total volume of the desired 5% alcohol solution.

910 mL = 0.0910 L

Step 2: Calculate the amount of 5% alcohol solution needed.

(0.0910 L) * (0.05) = 0.00455 L

Step 3: Calculate the amount of pure water needed.

(0.0910 L) - (0.00455 L) = 0.08645 L

Step 4: Calculate the amount of 65% alcohol mixture required.

(0.00455 L) / (0.65) = 0.007 L

Step 5: Calculate the amount of pure water in the 65% alcohol mixture.

(0.007 L) * (1 - 0.65) = 0.00245 L

Step 6: Calculate the total amount of pure water required.

(0.08645 L) + (0.00245 L) = 0.0889 L

To obtain the desired 910 mL of 5% alcohol solution, you will need 0.007 L of 65% alcohol mixture and 0.0889 L of pure water.

Dixie lives in a $67,000 wood house in the country whose contents are valued at $12,000. She wants to know her monthly property insurance premium. Use the table above to calculate her monthly property insurance premium.

Answers

Without the table indicating the stated premium rates and applicable discounts, the monthly property insurance cannot be computed. Hence, on the assumption that the rate is 2.5% and the discount applicable is 7.5% the premium per month will come to $152.24

What is Property Insurance?

The simple definition of Property Insurance is, it is an insurance policy that covers a policyholder for the structure of a building and its contents against stated or agreed perils.

What is the calculation for the above?

Given (based on assumption) that the applicable annual rate is 2.5%

and that:

Value of Wooden House = $67,000Value of the contents of the House = $12,000Applicable discount = 7.5% of premium/annum

Hence annual premium will be:

= (67,000 + 12000) * (2.5/100) * 0.925

Annual Premium Payable = $1,826.89.............A

Recall that we are asked to derive the monthly Insurance premium.

This is thus given as: Answer in A/12

→  1,826.89/12

= $152.24

Learn more about Insurance premiums at:

https://brainly.com/question/1191977

Bradley went to the store to buy ingredients for a new recipe. Artichokes were on sale for $3 per pound.
How much did Bradley pay if he bought
2
3
of a pound?
A $6. B $5. C $3 D $2

Answers

Answer :

Step-by-step explanation to problem:

2/3 * 3 = 2

we do 2/3 times 3 because $3 is for 1 pound and here we only need 2/3 of a pound

$2

Correct Answer = D

Can you help me, please!?

Answers

Answer:

The correct answer is (b+2)

Step-by-step explanation:

(3b-7) + (-2b+9)

3b-7-2b+9

3b-2b+9-7

b+2 Ans

Answer:

b+2 is the answer of this expression.

Step-by-step explanation:

(3b-7) and (-2b+9)

3b+(-2b) + (-7+9)

(3b-2b) + 2

b+2

Victor has been saving up money to buy the new iPhone 14 Pro Max , which retails for $ 1099 . He currently has from Lunar New Year , and started working as a tutor for $ 50 a week . The amount of money he has in his savings account can be modeled by V(w) = 50w + 500 where V(w) is the amount he has for w weeks of working as a tuto State the value of the slope and y - intercept . a b . What is the meaning of the slope and y - intercept ? How many weeks will he have to work to buy the new iPhone 14 Pro Max ?

Answers

For Victor to have enough money to purchase the new iPhone 14 Pro Max, he will need to work for roughly 12 weeks.

Is it an equation or an expression?

An expression is made up of a number, a variable, or a combination of a number, a variable, and operation symbols. An equation is created by joining two expressions with an equal sign. For illustration: When you add 8 and 3, you get 11.

a) Here is the equation for Victor's savings account:

V(w) = 50w + 500

Slope (m) = 50

Y-intercept (b) = 500

b) The slope (m = 50) represents the rate at which Victor is saving money per week. For every week that he works as a tutor, he saves $50. The y-intercept (b = 500) represents the initial amount of money that Victor had saved up before he started working as a tutor.

(c)To find out how many weeks Victor will have to work to buy the new iPhone 14 Pro Max, we need to set up an equation where the amount of money he has saved up (V(w)) is equal to the price of the phone ($1099). Therefore, we can write:

50w + 500 = 1099

Subtracting 500 from both sides, we get:

50w = 599

Dividing both sides by 50, we get:

w = 11.98

To know more about work visit:-

https://brainly.com/question/30265549

#SPJ1

PLEASE HELP

The linear function f(x) = 0.9× + 79 represents the average test score in your math class, where x is the number of the test taken. The linear function g(x) represents the average test score in your science class, where x is the number of the test taken.

Answers

The required answers are 80.8, 79,and g(42) > f(42).

How to find average of equation?

Part A:

To determine the test average for the math class after completing test 2, we need to evaluate the function f(x) at x=2. That is,

[tex]$$f(2) = 0.9(2) + 79 = 80.8$$[/tex]

Therefore, the test average for the math class after completing test 2 is 80.8.

Part B:

To determine the test average for the science class after completing test 2, we need to find the equation of the linear function g(x) that passes through the given points (1,78) and (2,79). The slope of the line passing through these points is

[tex]$m=\frac{y_2-y_1}{x_2-x_1}=\frac{79-78}{2-1}=1$$[/tex]

We can use the point-slope form of a line to find the equation of the line passing through the point (1,78) with slope m=1. That is,

[tex]$$y-78 = 1(x-1)$$[/tex]

Simplifying, we get

y = x + 77

Therefore, the test average for the science class after completing test 2 is

g(2) = 2 + 77 = 79

Part C:

To determine which class had a higher average after completing test 42, we need to evaluate f(42) and g(42) and compare the results. We have

[tex]$$f(42) = 0.9(42) + 79 = 117.8$$[/tex]

To find (42), we need to extend the linear function g(x) beyond the given data points by assuming that the function is linear and continues with the same slope m=1. That is,

g(x) = x + 77

for all [tex]$x\geq 1$[/tex]. Therefore,

[tex]$$g(42) = 42 + 77 = 119$$[/tex]

Since g(42) > f(42), we conclude that the science class had a higher average after completing test 42.

To know more Equation visit:

brainly.com/question/28243079

#SPJ1

consider the funciton f given by f(t)=log49(t7). find values for a and b such that f(t)=a+logb(t).

Answers

The values for a and b such that f(t) = a + logb(t) are,

a = log(1/log(49))

b = 49

Therefor

We can start by simplifying the expression for f(t)

f(t) = log49(t7)

f(t) = log(t7)/log(49) [Using the change of base formula]

Now we can rewrite f(t) as:

f(t) = log(t)/log(b) + a [Using the logarithmic properties log(ab) = log(a) + log(b)]

Comparing this expression with f(t) = a + logb(t), we can see that:

a = log(1/log(49)) [since log(b) = 1/log(49)]

b = t

Therefore, we have

f(t) = log(t7)/log(49) = log(t)/log(b) + log(1/log(49))

= log(t)/log(49) + log(1/log(49))

= log(1/log(49)) + log(t)/log(49)

So, a = log(1/log(49)) and b = 49.

Learn more about logarithmic property here

brainly.com/question/30226560

#SPJ4

4.1 h(x) Consider h(c) = cos 2x 4.1.1 Complete the table below, rounding your answer off to the first decimal where needed: -90° -75° -60° -45° -30° -15° 0° 15° 30° 45° 60° 75° 90° 4.1.2 Now use the table and draw the graph of h(x) = cos 2x on the system of axes below: -90°-75°-60-45-30-15 2- 14 - 1+ -24 (2) 15° 30° 45° 60⁰ 75⁰ 90⁰ (2) (2)​

Answers

Here's the completed table and the graph:

x  h(x)

-90°  1.0

-75°  -0.5

-60°  -1.0

-45° -0.0

-30° 1.0

-15° 0.5

0° 1.0

15° 0.5

30° -0.0

45° -1.0

60° -0.5

75° 1.0

90° 1.0

What is function?

In mathematics, a function is a relation between a set of inputs and a set of possible outputs, with the property that each input is related to exactly one output. Functions are often represented as a set of ordered pairs, where the first element of each pair is an input and the second element is the corresponding output. Functions are a fundamental concept in many areas of mathematics and have many real-world applications, including in science, engineering, and economics.

Here,

To calculate the values of h(c) in the table, we plug in the given values of x into the function h(c) = cos 2x and evaluate. For example, to find h(c) when x = -75°:

h(c) = cos 2x

h(c) = cos 2(-75°) (substitute -75° for x)

h(c) = cos (-150°) (simplify using the double angle identity)

h(c) = -0.5 (evaluate using the unit circle or a calculator)

We repeat this process for each value of x to fill out the table.

To graph the function h(x) = cos 2x, we plot each point from the table on the given system of axes. The x-axis represents the angle x in degrees, and the y-axis represents the value of h(x) = cos 2x. We then connect the points with a smooth curve to obtain the graph.

To know more about function,

https://brainly.com/question/28278699

#SPJ1

Other Questions
a burette has an initial reading of 3.50 ml. an experimenter opens the stopcock and after some time closes it recording a new burette reading of 12.75 ml. the experimenter repeats this two more times recording new readings of 15.60 ml and 17.25 ml. what is the total ml of titrant that has been dispensed from the burette. During a fight-or-flight response, epinephrine is released into the bodys circulatory system and transported throughout the body. Some cells exhibit a response to the epinephrine while other cells do not.Which of the following justifies the claim that differences in components of cell signaling pathways explain the different responses to epinephrine?a. Cell signaling depends on the ability to detect a signal molecule. Not all cells have receptors for epinephrine. Only cells with such receptors are capable of responding.b. Cell signaling depends on the transduction of a received signal by the nervous system. Not all cells are close enough to a synapse to receive the signal and respond.c. Cell signaling depends on the signal being able to diffuse through the cell membrane. Epinephrine is incapable of diffusing through some plasma membranes because of the membranes phospholipid composition.d. Cell signaling requires reception, transduction, and response. All cells can receive epinephrine, all cells respond with a pathway, but only select cells have the proper coding in their DNA to respond. after the civil war, southern states still controlled setting voter qualifications which permitted the creation of laws severely limiting and excluding black americans from voting. srs of 25 seniors 17 respond "yes" confidence interval for true proportion of seniors which of the following conditions were not met? Ain) is a special value that cannot be mistaken as a member of a list of data items and signals that there are no more data items to be processed a. terminator b. accumulator c. sentinel d. delimiter where is the evidence ImagineXin the below is a missing value. If I were to run a median imputer on this set of data what would the returned value be?50,60,70,80,100,60,5000,x(It's okay to have to look up how to do this!) An. error 80 100 70 The features in a model.... None of these answers are correct Are always functions of each other Kecp the model validation process stable Are used as proxics for y-hatfy (that is yhat divided by y) Which of the below were discussed as being problems with the hold out method for validation? Outliers can skew the result Validation is sometimes too challengingK=3is not sufficiently large cnough Data is not available for test and control differences. The modefis not trained on all of the day identify two or more challenges merchant ships faced a modern handball court is designed with __________ dimensions with a half court line. True or False: The great body of Gregorian chant is somewhat misattributed to Pope Gregory the Great due in part to some of the cultural practices and prejudices of the Middle Ages. Research on environmental influences on gender typing shows that teacherstend to negotiate more when boys misbehave, coming up with a joint plan to improve behavior.use more disapproval and controlling discipline with girls than with boys.seem to expect girls to misbehave more often than boys.give girls more encouragement than boys to participate in adult-structured activities. The fruits people like the most are shown in the circle graph.People who like different FruitsDates10%Bananas8%Other4%Grapes20%Apples34%peopleCherries24%If 750 people were surveyed, how many people like grapes? Enter the number of people in the box. Huda is experiencing a lot of cognitive development during adolescence. How is this BEST demonstrated in her life? A. She has experienced her first menstrual cycle and the growth of her breasts. B. She completely relies on her parents for most of her comfort and guidance. C. She has set a major goal for herself and has made some tough decisions. D. She is one of the last girls in her peer group to experience menarche. 1. Moralitatea este relativ din punct de vedere cultural, n consecin, nu este nimic n neregul cu circumcizia femeilor i nu ar trebui s o criticm In mr. Bunuelos class , 19 out of 26 student wore their school shirt of friday if the school has population of 2,462 student approximately how many students at the school wore their school shirt on friday? how does the amount of strokes on a magnet affect the strength of it? Note- I'm NOT a high schooler. Simple answers please. I'm 11. big wave entertainment is a newly-opened computer game developer. although the company creates quality computer games, no intermediary will agree to distribute their products. in order to get its games to consumers, big wave may need to create a(n) distribution system. Which of the following is the appropriate sequence for classically conditioning an eyeblink response to an auditory stimulus?a. deliver a puff of air to the eyelid, sound the toneb. sound the tone, deliver a puff of air to the eyelidc. follow the eyeblink response with a mild shockd. follow the eyeblink response with a soft drink Which literary element refers to the time and place indicated in a story? Malcolm X was assassinated. By members of the